University Calculus: Early Transcendentals (3rd Edition)

Published by Pearson
ISBN 10: 0321999584
ISBN 13: 978-0-32199-958-0

Chapter 9 - Section 9.1 - Sequences - Exercises - Page 489: 121

Answer

$\{a_{n}\}$ is convergent.

Work Step by Step

$a_{n}=\displaystyle \frac{1}{\sqrt{n}}+\sqrt{2}\geq\sqrt{2}$ $\{a_{n}\}$ is bounded below by $\sqrt{2}$ $\displaystyle \frac{1}{\sqrt{n}}$ is the greatest when n is the smallest (n=1) $\{a_{n}\}$ is bounded above by $\sqrt{2}+1$ $\{a_{n}\}$ is bounded. If we can show that it is monotonic, we can apply Th.6. \begin{align*} \sqrt{n+1}&\geq\sqrt{n} \\ \sqrt{n+1}+\sqrt{2n(n+1)}&\geq\sqrt{n} +\sqrt{2n(n+1)}\\ \displaystyle \frac{\sqrt{n+1}+\sqrt{2n(n+1)}}{\sqrt{n(n+1)}}&\displaystyle \geq\frac{\sqrt{n} +\sqrt{2n(n+1)}}{\sqrt{n(n+1)}} \\ \displaystyle \frac{1}{\sqrt{n}}+\sqrt{2}&\geq \displaystyle \frac{1}{\sqrt{n}+1}+\sqrt{2}\\ a_{n}&\geq a_{n+1} \\ \end{align*} $\{a_{n}\}$ is nonincreasing, that is, monotonic. Th.6 applies.
Update this answer!

You can help us out by revising, improving and updating this answer.

Update this answer

After you claim an answer you’ll have 24 hours to send in a draft. An editor will review the submission and either publish your submission or provide feedback.